Let $f : [1,2] → mathbb R$ be defined by $f(x) = x$. Prove that $f$ is Riemann integrable and compute $int_1^2f(x)dx$

The name of the pictureThe name of the pictureThe name of the pictureClash Royale CLAN TAG#URR8PPP











up vote
2
down vote

favorite












Let $f : [1,2] → mathbb R$ be defined by $f(x) = x$. Prove that $f$ is Riemann integrable and
compute $$int_1^2f(x)dx$$ as the limit of upper (and lower) sums.



I tried solving this but I don't know if my answer is right.



Let $P_n$ be the uniform partition of $[1,2]$ given by
$$1 lt 1+frac1n lt 1+frac2n lt cdots lt 1+fracn-1nlt2$$



The function $f(x)=x$ is increasing, hence
$$m_i=inf_x in [x_i-1,x_i]f(x)=f(x_i-1)=1+fraci-1n$$
and
$$M_i=sup_x in [x_i-1,x_i]f(x)=f(x_i)=1+fracin$$



$$L(f,P)=sum_1^n(1+fraci-1n)fracin=frac3n-12n$$ and $$U(f,P)=sum_1^n(1+fracin)fracin=frac3n+12n$$



Therefore $$lim_ntoinftyL(f,P_n)=frac32 quad lim_ntoinftyU(f,P_n)=frac32$$



By the Criterion of Integrability $$int_1^2f(x)dx=lim_ntoinftyL(f,P_n)=lim_ntoinftyU(f,P_n)=frac32$$







share|cite|improve this question






















  • The function is continuous then is Riemann integrable.
    – Nosrati
    Aug 16 at 9:55











  • yes but I had to prove as the limit of upper and lower sums
    – J.Dane
    Aug 16 at 9:58














up vote
2
down vote

favorite












Let $f : [1,2] → mathbb R$ be defined by $f(x) = x$. Prove that $f$ is Riemann integrable and
compute $$int_1^2f(x)dx$$ as the limit of upper (and lower) sums.



I tried solving this but I don't know if my answer is right.



Let $P_n$ be the uniform partition of $[1,2]$ given by
$$1 lt 1+frac1n lt 1+frac2n lt cdots lt 1+fracn-1nlt2$$



The function $f(x)=x$ is increasing, hence
$$m_i=inf_x in [x_i-1,x_i]f(x)=f(x_i-1)=1+fraci-1n$$
and
$$M_i=sup_x in [x_i-1,x_i]f(x)=f(x_i)=1+fracin$$



$$L(f,P)=sum_1^n(1+fraci-1n)fracin=frac3n-12n$$ and $$U(f,P)=sum_1^n(1+fracin)fracin=frac3n+12n$$



Therefore $$lim_ntoinftyL(f,P_n)=frac32 quad lim_ntoinftyU(f,P_n)=frac32$$



By the Criterion of Integrability $$int_1^2f(x)dx=lim_ntoinftyL(f,P_n)=lim_ntoinftyU(f,P_n)=frac32$$







share|cite|improve this question






















  • The function is continuous then is Riemann integrable.
    – Nosrati
    Aug 16 at 9:55











  • yes but I had to prove as the limit of upper and lower sums
    – J.Dane
    Aug 16 at 9:58












up vote
2
down vote

favorite









up vote
2
down vote

favorite











Let $f : [1,2] → mathbb R$ be defined by $f(x) = x$. Prove that $f$ is Riemann integrable and
compute $$int_1^2f(x)dx$$ as the limit of upper (and lower) sums.



I tried solving this but I don't know if my answer is right.



Let $P_n$ be the uniform partition of $[1,2]$ given by
$$1 lt 1+frac1n lt 1+frac2n lt cdots lt 1+fracn-1nlt2$$



The function $f(x)=x$ is increasing, hence
$$m_i=inf_x in [x_i-1,x_i]f(x)=f(x_i-1)=1+fraci-1n$$
and
$$M_i=sup_x in [x_i-1,x_i]f(x)=f(x_i)=1+fracin$$



$$L(f,P)=sum_1^n(1+fraci-1n)fracin=frac3n-12n$$ and $$U(f,P)=sum_1^n(1+fracin)fracin=frac3n+12n$$



Therefore $$lim_ntoinftyL(f,P_n)=frac32 quad lim_ntoinftyU(f,P_n)=frac32$$



By the Criterion of Integrability $$int_1^2f(x)dx=lim_ntoinftyL(f,P_n)=lim_ntoinftyU(f,P_n)=frac32$$







share|cite|improve this question














Let $f : [1,2] → mathbb R$ be defined by $f(x) = x$. Prove that $f$ is Riemann integrable and
compute $$int_1^2f(x)dx$$ as the limit of upper (and lower) sums.



I tried solving this but I don't know if my answer is right.



Let $P_n$ be the uniform partition of $[1,2]$ given by
$$1 lt 1+frac1n lt 1+frac2n lt cdots lt 1+fracn-1nlt2$$



The function $f(x)=x$ is increasing, hence
$$m_i=inf_x in [x_i-1,x_i]f(x)=f(x_i-1)=1+fraci-1n$$
and
$$M_i=sup_x in [x_i-1,x_i]f(x)=f(x_i)=1+fracin$$



$$L(f,P)=sum_1^n(1+fraci-1n)fracin=frac3n-12n$$ and $$U(f,P)=sum_1^n(1+fracin)fracin=frac3n+12n$$



Therefore $$lim_ntoinftyL(f,P_n)=frac32 quad lim_ntoinftyU(f,P_n)=frac32$$



By the Criterion of Integrability $$int_1^2f(x)dx=lim_ntoinftyL(f,P_n)=lim_ntoinftyU(f,P_n)=frac32$$









share|cite|improve this question













share|cite|improve this question




share|cite|improve this question








edited Aug 16 at 13:48









Jneven

575319




575319










asked Aug 16 at 9:44









J.Dane

187112




187112











  • The function is continuous then is Riemann integrable.
    – Nosrati
    Aug 16 at 9:55











  • yes but I had to prove as the limit of upper and lower sums
    – J.Dane
    Aug 16 at 9:58
















  • The function is continuous then is Riemann integrable.
    – Nosrati
    Aug 16 at 9:55











  • yes but I had to prove as the limit of upper and lower sums
    – J.Dane
    Aug 16 at 9:58















The function is continuous then is Riemann integrable.
– Nosrati
Aug 16 at 9:55





The function is continuous then is Riemann integrable.
– Nosrati
Aug 16 at 9:55













yes but I had to prove as the limit of upper and lower sums
– J.Dane
Aug 16 at 9:58




yes but I had to prove as the limit of upper and lower sums
– J.Dane
Aug 16 at 9:58










1 Answer
1






active

oldest

votes

















up vote
0
down vote













Yes, your proof is fine, everything is o.k.






share|cite|improve this answer




















  • Ok thanks, I wasn't sure when I took the values of $m_i$ and $M_i$
    – J.Dane
    Aug 16 at 9:49










  • Why the downvotes ?????????????
    – Fred
    Aug 16 at 9:53










  • The OP wrote: " I don't know if my answer is right." I said: "your answer (proof) is fine". Why the dowmvotes ?
    – Fred
    Aug 16 at 9:54










  • Is it really sufficient to consider uniform partitions only?
    – Martin R
    Aug 16 at 10:09










  • It is sufficient: we have $|U(f,P_n)-L(f,P_n)|= frac1n$. If $ epsilon >0$ the choose $n$ such that $ frac1n< epsilon$. Riemann's criterion shows then that $f$ is R- integrable. It follows then that $U(f,P_n) to int_1^2f(x)dx$.
    – Fred
    Aug 16 at 10:14










Your Answer




StackExchange.ifUsing("editor", function ()
return StackExchange.using("mathjaxEditing", function ()
StackExchange.MarkdownEditor.creationCallbacks.add(function (editor, postfix)
StackExchange.mathjaxEditing.prepareWmdForMathJax(editor, postfix, [["$", "$"], ["\\(","\\)"]]);
);
);
, "mathjax-editing");

StackExchange.ready(function()
var channelOptions =
tags: "".split(" "),
id: "69"
;
initTagRenderer("".split(" "), "".split(" "), channelOptions);

StackExchange.using("externalEditor", function()
// Have to fire editor after snippets, if snippets enabled
if (StackExchange.settings.snippets.snippetsEnabled)
StackExchange.using("snippets", function()
createEditor();
);

else
createEditor();

);

function createEditor()
StackExchange.prepareEditor(
heartbeatType: 'answer',
convertImagesToLinks: true,
noModals: false,
showLowRepImageUploadWarning: true,
reputationToPostImages: 10,
bindNavPrevention: true,
postfix: "",
noCode: true, onDemand: true,
discardSelector: ".discard-answer"
,immediatelyShowMarkdownHelp:true
);



);








 

draft saved


draft discarded


















StackExchange.ready(
function ()
StackExchange.openid.initPostLogin('.new-post-login', 'https%3a%2f%2fmath.stackexchange.com%2fquestions%2f2884603%2flet-f-1-2-%25e2%2586%2592-mathbb-r-be-defined-by-fx-x-prove-that-f-is-rieman%23new-answer', 'question_page');

);

Post as a guest






























1 Answer
1






active

oldest

votes








1 Answer
1






active

oldest

votes









active

oldest

votes






active

oldest

votes








up vote
0
down vote













Yes, your proof is fine, everything is o.k.






share|cite|improve this answer




















  • Ok thanks, I wasn't sure when I took the values of $m_i$ and $M_i$
    – J.Dane
    Aug 16 at 9:49










  • Why the downvotes ?????????????
    – Fred
    Aug 16 at 9:53










  • The OP wrote: " I don't know if my answer is right." I said: "your answer (proof) is fine". Why the dowmvotes ?
    – Fred
    Aug 16 at 9:54










  • Is it really sufficient to consider uniform partitions only?
    – Martin R
    Aug 16 at 10:09










  • It is sufficient: we have $|U(f,P_n)-L(f,P_n)|= frac1n$. If $ epsilon >0$ the choose $n$ such that $ frac1n< epsilon$. Riemann's criterion shows then that $f$ is R- integrable. It follows then that $U(f,P_n) to int_1^2f(x)dx$.
    – Fred
    Aug 16 at 10:14














up vote
0
down vote













Yes, your proof is fine, everything is o.k.






share|cite|improve this answer




















  • Ok thanks, I wasn't sure when I took the values of $m_i$ and $M_i$
    – J.Dane
    Aug 16 at 9:49










  • Why the downvotes ?????????????
    – Fred
    Aug 16 at 9:53










  • The OP wrote: " I don't know if my answer is right." I said: "your answer (proof) is fine". Why the dowmvotes ?
    – Fred
    Aug 16 at 9:54










  • Is it really sufficient to consider uniform partitions only?
    – Martin R
    Aug 16 at 10:09










  • It is sufficient: we have $|U(f,P_n)-L(f,P_n)|= frac1n$. If $ epsilon >0$ the choose $n$ such that $ frac1n< epsilon$. Riemann's criterion shows then that $f$ is R- integrable. It follows then that $U(f,P_n) to int_1^2f(x)dx$.
    – Fred
    Aug 16 at 10:14












up vote
0
down vote










up vote
0
down vote









Yes, your proof is fine, everything is o.k.






share|cite|improve this answer












Yes, your proof is fine, everything is o.k.







share|cite|improve this answer












share|cite|improve this answer



share|cite|improve this answer










answered Aug 16 at 9:48









Fred

38k1238




38k1238











  • Ok thanks, I wasn't sure when I took the values of $m_i$ and $M_i$
    – J.Dane
    Aug 16 at 9:49










  • Why the downvotes ?????????????
    – Fred
    Aug 16 at 9:53










  • The OP wrote: " I don't know if my answer is right." I said: "your answer (proof) is fine". Why the dowmvotes ?
    – Fred
    Aug 16 at 9:54










  • Is it really sufficient to consider uniform partitions only?
    – Martin R
    Aug 16 at 10:09










  • It is sufficient: we have $|U(f,P_n)-L(f,P_n)|= frac1n$. If $ epsilon >0$ the choose $n$ such that $ frac1n< epsilon$. Riemann's criterion shows then that $f$ is R- integrable. It follows then that $U(f,P_n) to int_1^2f(x)dx$.
    – Fred
    Aug 16 at 10:14
















  • Ok thanks, I wasn't sure when I took the values of $m_i$ and $M_i$
    – J.Dane
    Aug 16 at 9:49










  • Why the downvotes ?????????????
    – Fred
    Aug 16 at 9:53










  • The OP wrote: " I don't know if my answer is right." I said: "your answer (proof) is fine". Why the dowmvotes ?
    – Fred
    Aug 16 at 9:54










  • Is it really sufficient to consider uniform partitions only?
    – Martin R
    Aug 16 at 10:09










  • It is sufficient: we have $|U(f,P_n)-L(f,P_n)|= frac1n$. If $ epsilon >0$ the choose $n$ such that $ frac1n< epsilon$. Riemann's criterion shows then that $f$ is R- integrable. It follows then that $U(f,P_n) to int_1^2f(x)dx$.
    – Fred
    Aug 16 at 10:14















Ok thanks, I wasn't sure when I took the values of $m_i$ and $M_i$
– J.Dane
Aug 16 at 9:49




Ok thanks, I wasn't sure when I took the values of $m_i$ and $M_i$
– J.Dane
Aug 16 at 9:49












Why the downvotes ?????????????
– Fred
Aug 16 at 9:53




Why the downvotes ?????????????
– Fred
Aug 16 at 9:53












The OP wrote: " I don't know if my answer is right." I said: "your answer (proof) is fine". Why the dowmvotes ?
– Fred
Aug 16 at 9:54




The OP wrote: " I don't know if my answer is right." I said: "your answer (proof) is fine". Why the dowmvotes ?
– Fred
Aug 16 at 9:54












Is it really sufficient to consider uniform partitions only?
– Martin R
Aug 16 at 10:09




Is it really sufficient to consider uniform partitions only?
– Martin R
Aug 16 at 10:09












It is sufficient: we have $|U(f,P_n)-L(f,P_n)|= frac1n$. If $ epsilon >0$ the choose $n$ such that $ frac1n< epsilon$. Riemann's criterion shows then that $f$ is R- integrable. It follows then that $U(f,P_n) to int_1^2f(x)dx$.
– Fred
Aug 16 at 10:14




It is sufficient: we have $|U(f,P_n)-L(f,P_n)|= frac1n$. If $ epsilon >0$ the choose $n$ such that $ frac1n< epsilon$. Riemann's criterion shows then that $f$ is R- integrable. It follows then that $U(f,P_n) to int_1^2f(x)dx$.
– Fred
Aug 16 at 10:14












 

draft saved


draft discarded


























 


draft saved


draft discarded














StackExchange.ready(
function ()
StackExchange.openid.initPostLogin('.new-post-login', 'https%3a%2f%2fmath.stackexchange.com%2fquestions%2f2884603%2flet-f-1-2-%25e2%2586%2592-mathbb-r-be-defined-by-fx-x-prove-that-f-is-rieman%23new-answer', 'question_page');

);

Post as a guest













































































這個網誌中的熱門文章

Is there any way to eliminate the singular point to solve this integral by hand or by approximations?

Why am i infinitely getting the same tweet with the Twitter Search API?

Carbon dioxide